You are on page 1of 58

CHAPTER 14

PROBLEMS
I - Valuation of Assets and Liabilities, Consideration Transferred, Goodwill and Bargain
Purchase
Tony Inc. acquires all of Jeramillo Co’s assets and liabilities on January 1, 20x5. Tony incurs
the following cost for the acquisition.
50,000shares of new Tony common stock, par value
P2/share, market value P80/share, issued to the former
stockholders of Jaramillo……………………………………………… P 4,000,000 Cash payment
Registration fees connected with issuing the new shares,
Paid in cash………………………………………………………………….. 500,000 Cash payment
Cash paid to former stockholders Jeramillo: there
were 200,000 shares of Jeramillo outstanding, and
Tony agreed to pay P90 in cash for each share of
outstanding Jeramillo stock…………………………………………. 18,000,000 Cash payment
Consulting fees paid to Philippine brokers, in cash………. 1,100,000 Cash payment

The balance sheets of both companies immediately prior to the acquisition are as follows:
Tony Inc. Jaramillo Co.
Assets Book Value Book Value Fair Value

Cash……………………………….
. P 25,000,000 P 90,000 P 90,000

Receivables……………………. 2,000,000 200,000 190,000

Inventories……………………. 20,000,000 8,110,000 7,000,000


Plant & equipment, net… 99,500,000 50,000,000 40,000,000

Trademarks…………………… 5,000,000 1,000,000 4,000,000


Total Assets P 151,500,000 P 59,400,000
Liabilities & Equity
Current liabilities…………. P 500,000 P 400,000 400,000
Long-term liabilities…….. 70,000,000 45,000,000 47,000,000
Common stock, par………. 2,000,000 1,000,000
APIC……………………………… 55,000,000 10,000,000
Retained earnings……….. 25,000,000 6,000,000
Treasury Stock……………… -1000000 -3,000,000
Total Liabilities & Equity P 151,500,000 P 59,400,000

In addition to the assets and liabilities already reported, Jaramillo has the following
previously unrecorded intangible assets that meet the requirements for capitalization:
Intangible asset Fair Value
Brand names……………………………………………………………. P 5,000,000
Secret formulas……………………………………………………….. 7,000,000
Required:
1. Prepare the journal entry or entries to record the acquisition on Tony’s books.
2. Assume the same information as above, but Jaramillo has an additional previously
unreported intangible that meets the requirements for capitalization: a
noncompetition agreement with a fair value of P10,000,000. All fair value
calculations have been double checked for accuracy and found to be correct. Prepare
the journal entry or entries to record the acquisition on Tony’s books.
3. Prepare Tony’s balance sheet for (1) and (2) above immediately following the merger.
4. Determine the following amounts immediately following the merger: (a) total assets;
(b) total liabilities; (c) additional paid-in capital (share premium); (d) retained
earnings (accumulated profit or loss); and (e) stockholders’ equity;

II – Valuation of Assets acquired and Liabilities assumed, Measurement of Consideration


Transferred, Change in Value of assets acquired, Pre acquisition Contingency, In-process
R&D

Sandy Corporation’s balance sheet at January 2, 20x5 is as follows:

Sandy
Dr (Cr)
Cash and Receivables……………………………………………. P200,000,000
Inventories…………………………………………………………… 600,000,000
Property, plant and equipment, net……………………. 7,500,000,000
Current Liabilities……………………………………………….. (400,000,000)
Long-term debt…………………………………………………… 7,200,000,000
Capital Stock………………………………………………………. (7,200,000)
Retained earnings………………………………………………. (25,000,000)
Accumulated other comprehensive income………. (5,000,000)

An analysis of Sandy’s assets and liabilities reveals that book values of some reported items
do not reflect their market values at the date of acquisition:
 Inventories are overvalued by P200,000,000
 Property, plant and equipment are overvalued P2,000,000,000
 Long-term debt is undervalued by P100,000,000

In addition, the following items are not currently reported on Sandy’s balance sheets:
 Customer contracts, valued P25,000,000
 Skilled work force, valued at P45,000,000
 In-process research and developments, valued at P300,000,000
 Potential contracts with prospective customers, valued at P15,000,000
 Sandy has not recorder expected future warranty liabilities with the
present value at P10,000,00

On January 2, 20x5, Velasco issues new stock with a market value of P700,000,000 to acquire
the assets and liabilities of Sandy.
Stock registration fees are P100,000,000 paid in cash.

Consulting, accounting, and legal fees connected with the merger are P150,000,000 paid in
cash. In addition, Velasco enters into earning contingency agreement, whereby Velasco will
pay the former shareholders of Sandy an additional amount if Sandy’s performance meets
certain minimum levels. The present value of the contingency is estimated at P50,000,000.

Required:
1. Prepare the journal entry or entries Velasco make to record the acquisitions.
2. Now assume that during March, 20x5, new information, comes in regarding the value
of Sandy’s property, plant and equipment at the date of acquisition. It is determined
that the property was acquired worth P1,500,000 less than previously estimated.
Make the entry to record this new information.

III – Assets and Liabilities Acquired, Goodwill and Bargain Purchase Gain, Contingent
Consideration, Changes in Contingent Consideration
Here are the pre-acquisition balance sheets of POP Company and Sicle Company on
December 31, 20x5:
Pop Co. Sicle Co.
Book Value Book Value Market Values
Current assets P 5,000,000 P 2,000,000 P 1,500,000
Investments 1,000,000 500,000 500,000
Land 10,000,000 5,000,000 6,000,000
Buildings (net) 40,000,000 25,000,000 16,000,000
Equipment (net) 25,000,000 10,000,000 2,000,000
Total assets P 81,000,000 P 42,500,000
Current liabilities P 4,000,000 P 1,500,000 1,500,000
Long-term liabilities 20,000,000 10,000,000 12,000,000
Common stocks, P10
par 5,000,000 1,000,000
Additional paid-in
capital 40,000,000 20,000,000
Retained earnings 12,000,000 10,000,000
Total liabilities &
equity P 81,000,000 P 42,500,000

In addition to the above Sicle Co. has identifiable tangibles with a fair value of P5,000,000
not recognize on its book but appropriately capitalize by Pop.

On January 1, 20x6 Pop issues 400,000 shares of its stock, with a par value of P10/share and
a market value of 100/share, to acquire Sicle Company’s assets and liabilities. Stock
registration fees are P1,100,000, paid in cash.

Required:
1. Present the journal entry that Pop makes to record the acquisition.
2. In relation to no. 1 requirement, determine the following: (a) total assets; (b)
liabilities; (c) additional paid-in capital (share premium); (d) retained earnings
(accumulated profit or loss); and (e) stockholders’ equity;
3. With the same requirements of (1) and (2) above, but now assume Pop instead
issued 100,000 shares of stocks for Sicle’s assets and liabilities, and registration cost
are P800,000, paid in cash.
4. Now assume that Pop issues 100,000 shares for all of Sicle’s shares, as in
requirement above, and Pop agrees to pay cash to Salt’s previous owners if the
combined earnings of Pepper and Salt exceed a certain threshold over the next 2
years. The expected value of the earnings contingency is P8,000,000. Prepare Pop’s
acquisition entry.
5. Assume the facts as in requirements (3). Before the contingency period is over, the
estimated value of the earnings contingency declines to P5,000,000, Prepare Pop’s
entry to reflect the change in value ]of the earnings contingency, if
(a) The value decline occurs with the measurement period i.e. June 30, 20x6, then
the amount further declines by P200,000 on August 1, 20x6;
(b) The value is due to events occurring subsequent acquisition.

IV – Consideration Transferred; Cash Plus Contingent Consideration

Pham Company acquired the assets (except for the cash) and assume the liabilities of Senn
Company on January 1, 20x4, paying P720,000 cash. Senn Company’s December 31,20x3,
balance sheet, reflecting both book values and for values, showed;

Book Value Fair Value


Accounts receivable (net) ………………………….. P 72,000 P 65,000
Inventory …………………………………………………… 86,000 99,000
Land………………………………………………………….. 110,000 162,000
Buildings (net)…………………………………………… 369,000 450,000
Equipment (net)……………………………………….. 237,000 288,000
Total……………………………………………………… P 874,000 P 1,064,000
Accounts payable…………………………………….. P 83,000 P 83,000
Note payable…………………………………………… 180,000 180,000
Common stock, P2 par value…………………… 153,000
Other contributed capital………………………… 229,000
Retained earnings…………………………………… 229,000
Total……………………………………………………. P874,000

As part of the negotiations, Pham Company agreed to pay the former stockholders of Senn
Company P 135,000 cash if the post combination earnings of the combined company (Pham}
reached certain levels during 20x4 and 20x5.

Required:
1. Record the journal entry on the books of Pham Company to record the acquisition on
January 1, 20x4. It is expected that the earnings target is likely to be met.
2. Assuming the earnings contingent is met, prepare the journal entry on Pham
Company’s books to settle the contingency on January 2, 20x6.
3. Assuming the earnings contingency is not met, prepare the necessary journal entry
on Pham Company’s books on January 2, 20x6.

V – Acquiree: Two Corporations

Effective December 31, 20x4, Zintel Corporation proposes to issue additional shares of its
common stock in exchange for all the assets and liabilities of Smith Corporation and Platz
Corporation, after which Smith and Platz will distribute the Zintel stock to their stockholders
in complete liquidation and dissolution. Balance sheets of each of the corporations
immediately prior to merger on December 31, 20x4, follow. The common stock exchange
ratio was negotiated to be 1:1 for both Smith and Platz.

Zintel Smith Platz


Current
Assets…………………………………….. P 1,600,000 P 350,000 P 12,000
Long Term assets (net)
……………………….. 5,700,000 1,890,000 98,000
Total…………………………………………………
…. P 7,300,000 P 2,240,000 P 110,000

Current liabilities………………………………. P 700,000 P 110,000 P 9,000


Long-term debt…………………………………. 1,100,000 430,000 61,000
Common stock, P5 par value……………. 2,500,000 700,000 20,000
Retained earnings…………………………….. 3,000,000 1,000,000 20,000
Total…………………………………………………
… P 7,300,000 2,240,000 P 110,000

Required: Prepare journal entries on Zintel’s book to record the combination. Assume the
following:
1. The identifiable asset and liabilities of Smith and Platz are all reflected on the balance
sheets [above], and their recorded amount are equal to their current fair values
except for long-term assets.
2. The fair value of Smith’s long-term assets exceed their book value by P 20,000, in the
fair value of Platz long-term assets exceed their book values by P 5,000. Zintel’s
common stocks traded actively and has a current market price of P15/share.

VI – Allocation of Purchase Price to Various Assets and Liabilities

Company S has a long-term marketable securities, assume the following scenarios:


Case A: Assume that P Company paid P 130,000 cash for 100% of the net assets of the S
Company.

S Company
Long-lived
Current Assets Assets Liabilities Net Assets
Book Value P 15,000 P 85,000 P 20,000 P 80,000
Fair Value 20,000 130,000 30,000 120,000

Case B: Assume that P Company paid P 110,000 cash for 100% of the net assets of the S
Company.

S Company
Long-lived
Current Assets Assets Liabilities Net Assets
Book Value P 15,000 P 85,000 P 20,000 P 80,000
Fair Value 30,000 80,000 20,000 90,000

Case C: Assume that P Company paid P 15,000 cash for 100% of the net assets of the S
Company.

S Company
Long-lived
Current Assets Assets Liabilities Net Assets
Book Value P 15,000 P 85,000 P 20,000 P 80,000
Fair Value 40,000 40,000 40,000 20,000

Required: Complete the following schedule by listing the amount that would be recorded on
P’s books.
Retained Earnings
Current Long-lived (Gain in income
Goodwill Assets Assets Liabilities statement)
Case A
Case B
Case C

VII – Consideration Transferred: Using Bonds

On January 1, 20x4, Perez Company acquired all the assets and assumed all the liabilities of
Stalton Company and merged Stalton into Perez. In exchange for the net assets of Stalton,
Perez gave its bonds payable with maturity value of P 600,000 as stated rate of 10%, interest
payable semi annually on June 30 and December 31, a maturity of January 1, 20x4, and yield
rate of 12% balance sheets for Perez and Stalton (as well as fair value data) on January 1,
20x4 were as follows:

Perez Stalton
Book Value Book Value Fair Value
Cash……………………………………………. P250,000 P 114,000 P 114,000
Receivables…………………………………. 352,700 150,000 135,000
Invetories…………………………………… 848,300 232,000 310,000
Land…………………………………………… 700,000 100,000 315,000
Buildings…………………………………….. 950,000 410,000 54,900
(325,000
Accumulated depreciation - buildings ) (170,500)
Equipment…………………………………. 262,750 136,450 39,450
Accumulated depreciation – (70,050
equipment ) (90,450) _________
Total Assets………………………………… P2,968,700 P 881,500 P 968,350
Current Liabilities…………………………. P 292,700 P 95300 P 95,300
Bond Payable, 8% due 1/1/20x9, interest
Payable 6/30 and 12/31………………… 300,000 260,000
Common Stock, P15 par value……….. 2,500,000
Common Stock, P5 par value…………. 236,500
Other contributed capital……………… 950,000 170,000
Retained earnings………………………… 526,000 75,700
Total equities……………………………….. P 2,968,700 P 881,500

Required: Prepare the journal entry on the books of Perez Company to record the
acquisition of Stalton Company’s assets and liabilities in exchange for the bonds.

VIII – Acquisition with In-process Research and Development

On June 30, 20x4, SS Company reported the following account balances:

Receivables …………………… P80,000 Current Liabilities ………………… P (10,000)


Inventories …………………… 70,000 Long-Term liabilities ……………. (50,000)
Building (net) ………………. 75,000 Common stocks …………………… (90,000)
Equipment (net) ………….. 25,000 Retained earnings ……………….. (100,000)
Total Assets …………………. P250,000 Total liabilities and equities …. P(250,000)

On June 30, 20x4, PP paid P300,000 cash for all assets and liabilities of SS, which will cease
to exist as a separate entity. In connection with the acquisition, PP paid P 10,000 in direct
combination costs and agreed to pay P50,000 to the former owner of the SS contingent on
meeting certain revenue goals during 20x7, PP estimated the present value of its probability
adjusted expected payment for the contingency at P15,000

In determining its offer, PP noted the following pertaining to SS:


a. It hold a building with a fair value P40,000 more than its book value.
b. It has developed a customer list appraised at P22,000, although it is not recorded in
its financial records.
c. It has research and development activity in process with an appraised fair value of
P30,000. However, the project has not yet reached technological feasibility and the
assets used in the activity have no alternative future use.
d. Book values for the receivables, inventory, equipment and liabilities approximate fair
value.
Required: Prepare PP’s accounting entry to record the combination with SS using the
acquisition method.

IX – Comprehensive Problem: Goodwill Computation with Contingent Consideration

On December 21, 20x4, Pure Corporation enters into a business combination by acquiring
the assets and assumed the liabilities of Saint Corporation in which Saint Corporation will be
dissolved. Pure consideration transferred consist of the following:
a. 30,000 unissued shares of its P10 par common stock, with a market value of P25 per
share.
b. P180,000 in long-term 8% notes payable, and
c. A contingent payment of P120,000 cash on January 1, 20x7, if the average income of
during the 2 year period of 20x5 to 20x6 exceeds P 300,000 per year. Pure estimates
that there is a 30% of chance or probability that the P 120,000 payment will be
required.

In addition, Pure pays the following at the time of the merger:


 Finders’ fee P 12,000
 Accounting fees, P 24,000
 Legal fees to arrange the business combination P 42,000
 Cost of SEC registration, including accounting and legal fees P 18,000
 Cost of printing and issuing stock certificates P 14,400
 Indirect costs of combining, including allocated overhead and executive salaries P
27,600

Balance sheet and fair value information for the two companies on December 31, 20x4,
immediately before the merger, is as follows:

Book Value Fair Value Book Value Fair Value


Cash………………………………… P 276,000 P 276,000 P 24,000 P 24,000
Receivables - net…………….. 96,000 96,000 48,000 48,000
Inventories……………………… 288,000 360,000 120,000 72,000
Land………………………………. 108,000 240,000 72,000 240,000
Buildings - net (10-year life) 480,000 720,000 240,000 360,000
Equipment - net ( 5-year life) 432,000 588,000 216,000 300,000
In-process research and
Development………………….. 0 0 0 60,000
Total Assets……………….. P1,680,000 P2,280,000 P 720,000 P1,104,000

Accounts payable…………… P 216,000 P 216,000 P 72,000 P 72,000


Other liabilities……………… 240,000 216,000 144,000 168,000
Common stock, P10 par….. 720,000 240,000
Additional paid-in capital… 240,000 192,000
Retained Earnings………….. 264,000 72,000
Total Liabilities and Equities P1,680,00 P 720,000
0

Required:
1. On December 31, 20x4:
a. Determine the amount of goodwill
b. Prepare the entries required in the books of the acquirer (Pure) in relation to the
acquisition of Saint Corporation.
c. Prepare the balance sheet immediately after the business combination.

2. Provisional Amount on Assets Acquired. Assume that the value of the buildings was
provisionally determined on December 31, 20x4. On August 1, 20x5, Pure
Corporation received the final value from the independent appraisal, the fair value at
acquisition date being P 384,000.
a. Determine the amount of goodwill
b. Prepare the required entry to reflect the adjustment, if any.

3. Cash Contingency based on Future Earnings with Measurement Date Rule. Assume
that on August 31, 20x5 because of improved information about facts and
circumstances that existed at the acquisition date, the contingent consideration was
revised to an expected value of P 60,000.
a. Determine the amount of goodwill
b. Prepare the required entry to reflect the adjustment. If any.
c. On November 1, 20x5, the probability value of the contingent consideration
amounted to P 48,000:
c.1. Determine the amount of goodwill
c.2. Prepare the required entry to reflect the adjustment. If any.
c.3. On December 15, 20x5, due to subsequent change not existing on the
acquisition date the expected value of the contingent consideration amounted
was revised to P 78,000
c.3.1. Determine the amount of goodwill
c.3.2. Prepare the required entry to reflect the adjustment. If any.
c.3.3. On January 1, 20x7, Saint’s average income in 20x5 is P 324,000 and
20x6 is P 312,000 which means that the target is met:
c.3.3.1. Determine the amount of goodwill
c.3.3.2. Prepare the required entry to reflect the adjustment. If any.

4. Cash Contingency – with Present Value based on Future Performance – Cash Flows.
Assuming that if Saint Corporation will generates cash flows from operations of P
360,000 or more in 20x5. Saint estimates that there is a 35% chance that the P
120,000 will be required. Saint uses an interest rate of 4 5 to incorporate the time
value of money.
a. Determine the amount of goodwill
b. Prepare the required entry to reflect the adjustment. If any.
c. On December 31, 20x5, Saint Corporation’s cash flow from operations
amounted to P 336,000, which means that it did not exceed the cash flows
from operations threshold of P 360,000, therefore, there is no cash payment
to be made to Saint Corporation:
c.1. Determine the amount of goodwill
c.2. Prepare the required entry if the target event does not occur.
d. Assume that instead of contingent payment of P 120,000 cash , an additional
cash payment would be made on January 1, 20x7, equal to twice the amount
by which average annual earnings of Saint Corporation exceed P30,000 per
year, prior to January 1, 20x7. Net income was P 78,000 in 20x5 and P 84,000
in 20x6. What is the entry to record the payment of the contingent
consideration?

5. Stock Contingency based on Future Performance – Earnings with Market Value of


Stock Given. In addition to the stock issue, Pure Corporation also agreed to issue
additional shares of common stock to the former stockholders of Saint Corporation if
the average post-combination earnings over the next two years (i.e… 20x5 and 20x6)
equalled or exceeded P 390,000. The additional 1,200 shares expected to be issued
are valued at P 18,000.
a. Determine the amount of goodwill
b. Prepare entries required in the books of the acquirer in relation to the
acquisition of Saint Corporation.
c. On January 1, 20x7, the target is met or contingent event happens, I,e..
average post-combination earnings over the next two years amounted to
P492,000. What is the journal entry to record the issuance of 1,200 additional
shares?

6. Stock Contingency based on Future Performance – Earnings with No Market Value


of Stock Given. In addition to the stock issue, Pure Corporation agreed to issue
6,000additional shares if the average income during the 2-year period of 20x5-20x6
exceeded P 96,000 per year. On January 1, 20x7, the average income amounted to
P132,000 (the contingent event occurs). What is the journal entry to record the
issuance of 6,000 additional shares?

7. Stock Contingency based on Future Stock Prices. In addition to the stock issue, Pure
Corporation agreed to issue additional shares on January 1, 20x7, to compensate for
any fall in the value of peso any fall in the value of Pure common stock below P25 per
share. The settlement would be to cure the deficiency by issuing added shares based
on their fair value on January 1, 20x7. The market price of the shares on January 1,
20x7, was P20. On January 1, 20x7, the contingent event happens since the fair value
per share fall below P25. What is the journal entry to record the issuance of 7,500
additional shares?

8. Stock Contingency with Present Value based on Future Stock Prices. In addition to
the stock issue, Pure Corporation agreed to a payment of sufficient shares of Pure
Corporation common stock to ensure a total value of P750,000 if the fair value per
share is less than P 25 on December 31, 20x5. Pure estimates that there is a 40%
probability that the 30,000 shares issued will have a market value of P 510,000 on
December 31, 20x5, and 60% probability that the market value of the 30,000 shares
will exceed P 750,000. Pure uses an interest rate of 4% to incorporate the time value
of money. On December 31, 20x5, the contingent event occurs, wherein Pure’s stock
rice had fallen to P 20, thus requiring Pure to issue additional shares of stock to the
former owners of Saint Corporation. What is the journal entry to record the issuance
of additional shares?

X – Liquidation of Acquiree, Accounting by Acquirer

Homer Ltd. Is seeking to expand its shares of the widgets market and has the negotiated to
take over the operations of Tan Ltd. on January 1, 20x4. The balance sheets of the two
companies as at December 31, 20x4 were as follows:

Homer Tan
Cash…………………………………………………
. P 23,000 P 12,000
Receivables…………………………………….. 25,000 34,700
Inventory………………………………………… 35,500 27,600
Freehold Land………………………………… 150,000 100,000
Buildings (net)……………………………….. 60,000 30,000
Plant and equipment (net)……………. 65,000 46,000
Goodwill………………………………………… 25,000 2,000
P 383,500 P 252,300

Accounts payable…………………………. P 56,000 P 43,500


Mortgage loan……………………………… 50,000 40,000
Debentures………………………………….. 100,000 50,000
Common Stock, 100,000 shares……. 100,000
Common Stock, 60,000 shares……… 60,000
Additional pais-in capital…………….. 28,500 26,800
Reained earnings…………………………. 49,000 32,000
P 383,500 P 252,300

Homer Ltd. Is to acquire all the assets, except cash of Tan Ltd. The assets of Tan are all
recorded at fair value except:

Fair Value
Inventory……………………………………………………………………………. P39,000
Freehold land……………………………………………………………………… 130,000
Buildings…………………………………………………………………………….. 40,000

In exchange, Homer Ltd. is to provide sufficient extra cash to allow Tan Ltd. to repay all of its
outstanding debts and its liquidation cost of P2,400, plus 2 fully-paid shares in Homer Ltd.
for every 3 shares held in Tan Ltd. The fair value of a share in Hastings Ltd. is P3.20. An
investigation by the liquidator of Tan Ltd. reveals that on December 31, 20x3, the following
outstanding but had not been recorded:

Accounts Payable……………………………… P 1,600


Mortgage Interest…………………………….. 4,000
The debentures issued by Tan Ltd. are to be redeemed at 5% premium. Cost of issuing the
shares were P1,200.
Required:
1. Prepare the acquisition analysis and journal entries to record the business
combination in the records of Homer Ltd.
2. Prepare the liquidation, liquidator’s cash, and shareholders’ distribution accounts for
Tan Ltd.
XI – Appendix Problem: Acquisition Entry, Deferred Taxes
Patel Company issued 100,000 shares of P1 par value common stock (market value of
P6/share) for the net assets of Seely Company on January 1, 20x4, in a statutory merger,
Seely Company had the following assets, liabilities, and owners’ equity at the time:
Book Value
Tax Basis Fair Value Difference
Cash P20,00 P20,000 P0
Accounts Receivable 112,000 112,000 0
Inventory 82,000 134,000 52,000
Land 30,000 55,000 25,000
Plant assets (net) 392,000 463,000 71,000
Total assets P636,000 P784,000 P626,000

Allowance for uncollectible accounts P10,000 P10,000 P0


Accounts payable 54,000 54,000 0
Bonds Payable 200,000 180,000 20,000
Common stock, P1 par value 80,000
Other contributed capital 132,000
Retained earnings 160,000
Total equities P636,000

Required: Prepare the journal entry to record the assets acquired and liabilities assumed.
Assume an income tax rate of 40%.

MULTIPLE CHOICE PROBLEMS


1. Manet Corporation exchange 150,000 shares of newly issued P1 par value common
stock with a fair market value of P25 per share for all of the outstanding P5 par value
common stock of Gardner Inc. and Gardner is then dissolved. Manet paid the
following cost and expenses related to the business combination.
Cost of special shareholder’ meeting to vote on the merger…….. P 13,000
Registering and issuing securities………………………………………………. 14,000
Accounting and legal fees………………………………………………………….. 9,000
Salaries of Manet’s employees assigned to the implementation
of the merger…………………………………………………………………………. 15,000
Cost of closing duplicate facilities……………………………………………… 11,000

In the business combination of Manet and Gardner:


a. All of the items listed above are treated as expense
b. All of the items listed above except the cost of registering and issuing the
securities are capitalized
c. The cost of registering and issuing the securities are deducted from the
fair market value of the common stock used to acquire Gardner
d. Only the cost of closing duplicate facilities, the salaries of Manet
employees is assigned to the merger, and the cost of the shareholders’
meeting would be treated as expenses
Items 2 and 4 are based on the following information:
Geri acquired the net assets of Caiga Corp. on July 1, 20x5. In exchange for net assets at fair
market value of Caiga Co. amounting to P835,740. Geri issued 81,600 shares of a market
price of P12 per share (P9 par value).
Out of pocket costs of the combination were as follows:
Legal fees for the contract of business combination……………………………. P 42,720
Audit fee for SEC registration of share issue…………………………………………. 108,000
Printing costs of share certificates………………………………………………………… 17,400
Broker’s fee…………………………………………………………………………………………... 28,320
Accountant’s fee for pre-acquisition audit…………………………………………….. 96,000
Other direct cost of acquisition………………………………………………………………. 90,000
General and allocated expenses…………………………………………………………….. 51,600
Listing fees in issuing new shares……………………………………………………………. 36,000
Documentary stamp tax on the new shares……………………………………………. 7,200
Geri will pay on additional cash consideration of P546,000 in the event that Caiga’s net
income will be equal or greater than P1,140,000 for the period ended December 31, 20x5.
At the acquisition, there is a high probability of reaching the target net income and the fair
value of the additional consideration was determined to P234,000. Actual net income for the
period ended December 31, 20x5 amounted to P1,500,000. The additional consideration
was paid.
2. What is the amount of goodwill to be recognized in the statement of financial
position as of December 31, 20x5?
a. P -0- c. P377,460
b. P257,040 d. P425,640

3. What amount of expense is to be recognized for the year ended December 31, 20x5,
assuming Geri issued 45,000 share capital on July 1, 20x5?
a. P -0- c. P377,640
b. P257,200 d. P620,640

4. What amount to be chargeable to operation/profit or loss (net) for the year ended
December 31, 20x5, assuming that the Geri issued 45,000 shares?
a. P -0- c. P315,720
b. P257,200 d. P558,900
Items 5 and 6 are based on the following information:
Balance sheet information for Hope Corporation at January 1, 20x4, a summarized as
follows:

Current Assets……………. P920,000 Liabilities………………… P1,200,000


Plant Asset……………….. 1,800,000 Capital Stock, P10 par 800,000
Retained Earnings…… 720,000
P2,720,000 P2,720,000

Hope’s assets and liabilities are fairly valued except for plant assets that are undervalued by
P200,000. On January 2, 20x4, Robin corporation issues 80,000 shares of P10 par value
common stock for all of Hope’s net assets and Hope is dissolved. Market quotations for the
two stocks on this date are:
Robin common: P28; Hope common: P19
Robin pays the following fees and costs in connection with the combination:
Finder’s fee, P10,000
Cost of registering and issuing stock, P5,000
Legal and accounting fees, P6,000
5. Calculate of the amount consideration transferred:
a. P2,240,000 c. P2,256,000
b. P2,251,000 d. None of the above

6. Calculate any goodwill from the business combination:


a. P475,000 c. P531,000
b. P520,000 d. None of the above

7. Maplewood Corporation purchase the net asset of West Corporation on January 2,


20x4 for P500,000 and also paid P20,000 in direct acquisition cost. West’s balance
sheet on January 1, 20x4 was as follows:

Accounts Receivable, net………………………………………..P180,000


Inventory……………………………………………………………….. 360,000
Land……………………………………………………………………….. 40,000
Building (net)………………………………………………………….. 60,000
Equipment (net)……………………………………………………… 80,000
Current Liabilities……………………………………………………. 70,000
Long-term debt………………………………………………………. 160,000
Common stock, P1 par……………………………………………. 20,000
Paid-in capital…………………………………………………………. 430,000
Retained Earnings…………………………………………………… 40,000

Fair value agrees with book values except for inventory land, and equipment, which
have fair value of P400,000, P50,000, and P70,000, respectively. West has patent
rights valued P20,000. The bargain purchase gain amounted to:
a. None c. P50,000
b. P30,000 d. P70,000

8. Pretzel Company acquire the assets (except for cash) and assume the liabilities of Salt
Company on January 2, 20x4. As compensation, Pretzel Company gave 30,000 shares
of its common stock, 15,000 shares of its 3% preferred stock and cash of P50,000 to
the stockholder of Salt Company. On the acquisition date, Pretzel Company stock
had the following characteristics:
Pretzel Company
Stock Par Value Fair Value
Common…………………………………… P10 P25
Preferred………………………………….. 100 100

Immediately prior to the acquisition, Salt Company’s balance sheet reported the
following book values and fair values:

Book Value Fair Value


Cash……………………………………………………………… P165,000 P165,000
Accounts receivable(net of P11,000 allowance) 220,000 198,000
Inventory – FiFO cost……………………………………. 275,000 330,000
Land……………………………………………………………… 396,000 550,000
Buildings and equipment (net)……………………... 1,144,000 1,144,000
Current Liabilities…………………………………………. 275,000 275,000
Bonds Payable, 10%........................................ 450,000 495,000
Common stock, P5 par value……………………….. 770,000
Other contributed capital…………………………….. 396,000
Retained Earnings…………………………………………. 309,000

Calculate any goodwill from the business combination.


a. P -0- c. P798,000
b. P683,000 d. P848,000

9. Pritano Company acquired at the net assets of Succo Company on December 31,
20x4, for P2,160,000 cash. The balance sheet of Succo Company immediately prior to
the acquisition showed:

Book Value Fair Value


Current Assets…………………………………… P960,000 P960,000
Plant and Equipment………………………… 1,080,000 1,440,000
Total…………………………………………………. P2,040,000 P2,400,000
Liabilities………………………………………….. P180,000 216,000
Common stock…………………………………. 480,000
Other contributed capital………………… 600,000
Retained earnings……………………………. 780,000
Total………………………………………………… 2,040,000

As part of the negotiations, Pritano agreed to pay the stockholders of Succo


P360,000 cash if the post-combination earnings of Pritano averaged P2,160,000 or
more per year over the next two years.
Calculate at any goodwill from the business combination
a. P24,000 c. P324,000
b. P36,000 d. P336,000
10. Air Philippines June 1, 20x5 balance sheet is as follows (in millions):

Assets Liabilities and equity


Cash P1,400 Current liabilities P3,200
Receivables 650 Long-term debt 5,000
Investments 1000 Common stock, P.01 par 1
Maintenance Supplies 150 Additional paid-in capital 5,500
Flight equipment (net of P2,000 8,500 Retained earnings
accumulated depreciation) (deficit) (2,300)
Accumulated other
International routes 700 comprehensive income 1,999
Treasury stock (1,000)
Total P12,400 P12,400

Philippine Airlines acquired Air Philippines on June 1, 20x5. Philippine Airlines


accounted for the acquisition by putting Air Philippines’ assets and liabilities directly
on its own books. Air Philippines’ cash and receivables, investments, and current
liabilities were reported at market value. Its maintenance supplies had a fair value of
P400 million, flight equipment had a fair value of P12,000 million, and international
routes were worth P500 million. Long-term debt had a fair value of P6,000 million.
Air Philippines also had an unrecorded intangible, representing leases with
favourable terms, worth P800 million. Philippine Airlines paid P8,000 million in cash
for Air Philippines. The gain/goodwill arising from the business combination (in
millions):
a. P1,250 gain c. P450 goodwill
b. P1,650 gain d. P850 goodwill
Items 11 and 12 are based on the following information:
On January 1, 20x4, Platz Company acquired all the net assets Satz Company by issuing
75,000 shares of its P10 par value common stock to the stockholders of Satz Company.
During negotiation, Platz Company agreed to issue additional shares of common stock to the
stockholders of Satz if the average post combination earnings over the next three years
equalled or exceeded P2,500,000. On January 1, 20x4, the market value of Platz stock was
P50 per share. Based on the information available of the acquisition date, the additional
10,000 shares are expected to be issued.
11. Calculate any goodwill from the business combination on Platz Company’s books on
January 1, 20x4. It is expected that the earnings target is likely to be met.
a. P0 c. P500,000
b. P100,000 d. Not determinable

12. Calculate the amount of paid-in capital in excess of par to be credited arising from
meeting the target on January 1, 20x7, when the additional shares are issued, on this
date the market value of Platz stock is valued at P60 per share.
a. P0 c. P400,000
b. P100,000 d. P500,000

13. Edina Company acquired the assets (except cash) and assumed the liabilities of Burns
Company on January 1, 20x4, paying P2,600,000 cash. Immediately prior to the
acquisition Burns Company’s balance sheet was as follows:

Book Value Fair Value


Accounts receivable (net) P240,000 P220,000
Inventory 290,000 320,000
Land 960,000 1,508,000
Buildings (net) 1,020,000 1,392,000
Total P2,510,000 P3,440,000
Accounts payable P270,000 P270,000
Note payable 600,000 600,000
Common stock, P5 par 420,000
Other contributed capital 640,000
Retained earnings 580,000
Total P2,510,000
Edina Company agreed to pay Burn Company’s former stockholders P200,000 cash in
20x6 if post-combination earnings of the combined company reached P1,100,000
during 20x5.
Calculate the gain on contingent consideration for Edina Company in 20x6 assuming
the earnings contingency was not met:
a. P0 c. P200,000
b. P30,000 d. P230,000
Items 14 to 16 are based on the following information:
On January 1, 20x5, CC Co. acquired the identifiable net asset of DD, Inc. On this date, the
identifiable assets acquired and liabilities assumed have fair values of P7,680,000 and
P4,320,000, respectively. CC Co. incurred the following acquisition-related costs: legal fees,
P48,000, due diligence costs, P480,000; and general and administrative costs of maintaining
an iternal acquisition , P96,000. As considerations, CC Co. transferred 9600 of its own shares
with par value and fair value per share of P400 and P500, respectively, to DD’s former
owners. Costs of registering the shares (previously issued and newly issued) amounted to
P192,000 (P24,000 pertains to listing fees of previously issued shares)
14. How much is the goodwill (gain on bargain purchase) on the business combinations?
a. P667,200 c. P1,440,000
b. P720,000 d. None of the above

15. How much is the total amount charged to profit or loss in relation to the transaction
above?
a. P624,200 c. P816,000
b. P648,000 d. None of the above

16. Ignoring the consideration and issue costs above, but instead, CC Co. issued bonds
with face value and fair value of P4,800,000 before incurring the transaction costs.
Transaction costs in issuing the bonds amounted to P240,000. How much is the
goodwill (gain on bargain purchase) on the business combination?
a. P667,200 c.P1,440,000
b. P720,000 d. None of the above
Items 17 and 18 are based on the following information:
On January 1, 20x5, Kim Co. acquired all of the identifiable assets and assumed all liabilities
of Dorothy Inc. by paying cash of P4,800,000. On this date, identifiable assets and liabilities
assumed have fair value of P7,680,000 and P4,320,000, respectively. Kim has estimated
restructuring provisions of P960,000 representing exit costs of the acquiree’s activities,
termination costs of employees of Dorothy and relocation costs of the said employees. The
restructuring plan is conditional until the business combination process is done if the
combination will not happen, no restructuring will happen.
17. For purposes of computing the goodwill (gain on bargain purchase), How much is the
fair value of net assets to be deducted from the consideration transferred?
a. P2,400,000 c. P5,280,000
b. P3,360,000 d. None of the above

18. How much is the goodwill (gain on bargain purchase) on the business combination:
a. (P480,000) c. P2,400,000
b. P1,440,000 d. None of the above
19. On January 1, 20x5, Drei Co. acquired all of the identifiable assets and assumed all
liabilities of Cerise Inc. by paying P4,800,000. On this date, identifiable assets and
liabilities assumed have fair value of P7,680,000 and P4,320,000, respectively. Terms
of agreement are as follows: (a) 20% of the price shall be paid on January 1, 20x5 and
the balance on December 31, 20x6 (the prevailing market rate on the same date is
10%): (b) the acquirer shall also transfer its piece of land with book and fair value of
P2,400,000 and P1,440,000 respectively. Included in the liabilities assumed is an
estimated warranty liability. The carrying amount and fair value of this warranty
liability amounted to P576,000 and P468,000, respectively. The acquiree guarantees
that the warranty liability would be only be settled for P480,000. How much is the
goodwill on the business combination?

a. P2,105,376 c. P2,213,376
b. P2,201,376 d. None of the above
Items 20 to 24 are based on the following information:
On January 1, 20x4, NT Company exchanges 15,000 shares of its common stock for all of the
assets and liabilities of OTG Inc. Each of NT’s shares has a P4 par value and a P50 fair value.
The fair value of the stock exchanged in the acquisition was considered equal to OTG’s fair
value. NT also paid P25,000 in stock registration and issuance costs in connection with the
merger.
Several of OTG’s accounts have fair values that differ from their book values on the date:

Book Values Fair Values


Receivables P65,000 P63,000
Trademarks 95,000 225,000
Record music catalog 60,000 180,000
In-process research and development 0 200,000
Notes payable 50,000 45,000

Pre-combination January 1, 20x4, book values for the two companies are as follows:

NT OTG
Cash P60,000 P29,000
Receivables 150,000 65,000
Trademarks 400,000 95,000
Record music catalog 840,000 60,000
Equipment (net) 320,000 105,000
Total P1,770,000 P354,000
Accounts payable P110,000 P34,000
Notes payable 370,000 50,000
Common stock 400,000 50,000
Additional paid-in capital 30,000 30,000
Retained earnings 860,000 190,000
Total P1,770,000 P354,000

Assume that this combination is a statutory merger so that OTG’s account will be transferred
to the record of NT. OTG will be dissolved and will no longer exist as a legal entity.
Immediately business combination using the acquisition method, determine:
20. The total assets amounted to:
a. P2,214,000 c. P2,574,000
b. P2,547,000 d. P2,599,000

21. The total liabilities amounted to:


a. P84,000 c. P564,000
b. P480,000 d. P559,000

22. The common stock amounted to:


a. P50,000 c. P450,000
b. P400,000 d. P460,000

23. The additional paid-in capital amounted to:


a. P30,000 c. P695,000
b. P60,000 d. P720,000

24. The retained earnings amounted to:


a. P190,000 c. P860,000
b. P835,000 d, P1,050,000
Items 25 to 31 are based on the following information:
On December 31, 20x4, PP Inc., acquired assets and liabilities of SS Company. PP will
maintain SS as a wholly owned subsidiary with its own legal and accounting identity. The
consideration transferred to the owner of SS included 50,000 newly issued PP common
shares (P20 market value, P5 par value) and an agreement to pay an additional P130,000
cash if SS meets certain project completion goals by December 31, 20x5. PP estimates 50%
probability that SS will be successful in meeting these goals and uses a 4% discount rate to
represent time value of money.
Immediately prior to the acquisition, the following data for both terms were available:

SS SS
PP Book Values Fair Values
Revenues (P1,200,000)
Expenses 875,000
Net Income P325,000
Retained earnings, 1/1/20x4 (950,000)
Net income (325,000)
Dividends paid 90,000
Retained earnings, 12/31/20x4 (P1,185,000)
Cash P85,000 P85,000
Receivables and inventory 190,000 180,000
Property, plant, and equipment 450,000 600,000
Trademarks 300,000 160,000 200,000
Total Assets P2,560,000 P885,000
Liabilities (P500,000) (P180,000) (P180,000)
Additional paid-in capital (400,000) (200,000)
Common stock (475,000) (70,000)
Retained earnings (1,185,000) (435,000)
Total liabilities and equities (P2,560,000) P885,000

Note: Parenthesis a credit balance.


In addition, PP assessed a research and development project under way at SS to have a fair
value of P100,000. PP paid legal and accounting fees of P15,000 in connection with the
acquisition and P9,000 stock issue and registration costs. Use a 0.961538 present value
factor where applicable.
25. The consideration transferred amounted to:
a. P1,000,000 c. P1,030,000
b. P1,015,000 d. P1,062,500

26. The additional paid-in capital after combination amounted to:


a. P400,000 c. P1,141,000
b. P600,000 d. P884,000

27. The expenses for 20x4 amounted to:


a. P0 c. P884,000
b. P875,000 d. P890,000

28. The net income for 20x4 amounted to:


a. P0 c. P316,000
b. P310,000 d. P325,000

29. The retained earnings amounted to:


a. P435,000 c. P1,185,000
b. P1,170,000 d. P1,620,000
30. Assuming that on June 15,20x5, the contingent performance obligation was revised
to P75,000 due to facts and information that exists on December 31, 20x4, determine
the amount of goodwill?
a. P0 c. P75,000
b. P62,500 D. P90,000

31. In relation to No. 30, assuming that on July 31,20x6, the contingent performance
obligation was revised to P80,000 due to facts and information that exists on
December 31, 20x4, determine the amount of goodwill and contingent performance
obligation?
Goodwill Obligation Goodwill Obligation
a. P90,000 P75,000 c. P95,000 P75,000
b. P90,000 P80,000 d. P95,000 P80,000

32. To induce the owners of Axel Company to sell to Ayala Corporation, an earnout was
included in the acquisition agreement. Ayala agrees to pay the former owners of Axel
P5.00 for every peso of total net income before interest and taxes (NBIT) earned over
P20 million in the next four years. The payment would be made at the end of four
years. Expected total NBIT in the next four years is as follows?

Total NBIT (earned) Probability


P5,000,000 0.20
15,000,000 0.50
30,000,000 0.20
35,000,000 0.10

What is the value of the earnout at the date of acquisition, assuming a discount rate
of 12% (PV factor of 1.57351936)?
a. P11,121,566 c. P15,751,936
b. P11,801,935 d. 17,500,000

33. Raphael Company paid P20,000,000 for the net assets of Paris Corporation and Paris
was then dissolved. Paris had no liabilities. The fair values of Paris’ assets P2,500,000.
Paris only non-current assets were land and equipment with fair values of P160, were
000 and P640,000, respectively. At what value will the equipment be recorded by
Raphael?
a. P640,000 c. P240,000
b. P400,000 d. P-0-

34. Company Y is purchased by Company X, and the purchase price is P2,500,000 greater
than the fair values of the identifiable net assets acquired. One of the assets acquired
is a building, originally valued at P1,000,000 at the date of the purchase. Six months
after the acquisition, it is discovered that the building was really only worth P200,000
at the date of acquisition. What entry is made to reflect this new information?
a. dr. goodwill, cr. building for P800,000
b. dr. loss on building, cr. building for P800,000
c. dr. other contributed capital, cr. building for P800,000
d. dr. retained earnings, cr. building for P800,000

35. Bolton Company acquires the net assets of Pamela Company for a cash consideration
of P100,000. One half is to be paid on acquisition date and one half is payable in one
year’s time. The appropriate discount rate is 10% p.a. The present value of the cash
outflow in one year’s time is:
a. P45,454 c. P54,545
b. P50,000 d. P55,000

36. On October 1, 20x4, the Tingling Company acquired the net assets of the Greenbank
Company when the fair value of Greenbank’s net assets was P116 million and their
carrying amount was P120 million. The consideration transferred comprised P200
million in cash transferred amount at the acquisition date, plus another P60 million in
cash to be transferred 11 months after the acquisition date if a specified profit target
was met by Greenbank. At the acquisition date there was only a low probability of
the profit target being met , so the fair value of the additional consideration liability
was P10 million . In the event, the profit target was meta and the P60 million cash
was transferred.
What amount should Tingling present for goodwill in its statement of consolidated
financial position on December 31, 20x4, according to PFRS3 Business combinations?
a. P80 million c. P94 million
b. P84 million d. P144 million

37. An acquirer made the following entry to report on acquisitions:


Tangible assets……………………………………4,000
Customer lists…………………………………….. 600
Goodwill…………………………………………….. 1,000
Cash……………………………………………………....2,000
Liabilities…………………………………………………3,600
Six months after the acquisition, the customers lists are determined to be worthless.
How is this information reported if (1) the new information relates to the value of the
customer lists as of the date of acquisition, and (2) the new information relates to
changes in value since acquisition? Customer lists are written off, and
(1) (2)
a. A gain on acquisition of P600 is recordedGoodwill decreases P600
b. Goodwill increases P600 A loss of P600 is recorded
c. A loss of P600 is recorded Goodwill increases P600
d. Cash is reduced by P600 A loss of P600 is recorded
38. Dosmann Inc., acquired net assets of Lizzi Corporation on January 1, 20x4, for
P700,000 in cash. This portion of the consideration transferred results in a fair-
valueallocation of P35,000 to equipment and goodwill of P88,000. At the acquisition
date, Dosmann also agrees to pay Lizzi’s previous owners an additional P110,000 on
January 1, 20x6, if Lizzi earns a 10% return on the fair value of its assets in 20x4 and
20x5. Lizzi’s profits exceed this threshold in both years. Which of the following is
true?
a. The additional P110,000 payment is a reduction in retained earnings.
b. The fair value of the expected contingent payment increases goodwill at the
acquisition date,
c. Goodwill as of January 1, 20x6, increases by P110,000
d. The P110,000 is recorded as an expense in 20x6
Use the following information to answer question 39-42
The balance sheet of Salt Company, along the market values of its assets and liabilities, is as
follows:

Salt Company
Book Value Market Value
dr (cr) dr (cr)
Current Assets P2,000,000 P1,500,000
Plant and equipment (net) 30,000,000 35,000,000
Patents 100,000 2,000,000
Complete technology 0 10,000,000
Broader customer based 0 16,000,000
Technically skilled workforce 3,000,000
Potentially profitable future contract 2,000,000
Licensing agreement 0 4,000,000
Potential contracts with new customers 1,500,000
Advertising jingles 1,000,000
Future cost savings 1,800,000
Goodwill 200,000 700,000
Liabilities (28,000,000) (30,000,000)
Common stock, P10 par (1,000,000)
Additional paid-in capital (5,000,000)
Retained earnings 1,700,000

39. Pail Company pays P100,000,000 in cash for Sal t Company’s assets and liabilities. Pail
records goodwill of:
a. P50,800,000 c. P72,500,000
b. P66,800,000 d. P77,500,00

40. Now assume Pail Company pays P10,000,000 in cash to acquire the assets and
liabilities of Salt Company. Pail records a bargain purchase gain on acquisition of:
a. Zero c. P17,500,000
b. P132,500,000 d. P28,500,000

41. Pail paid P100,000,000 in cash for Salt. Three months later, Salt’s patents are
determined to have been worthless as of the date of acquisition. The entry to record
this information includes
a. A debit to loss of P2,000,000
b. A debits to patents of P2,000,000
c. A debit to goodwill of P2,000,000
d. A debit to retained earnings of P2,000,000

42. Pail paid P10,000,000 in cash for Seattle. Three months later, it is determined that
Seattles’s acquisition-date liabilities omitted a pending lawsuit valued at P2,000,000.
The entry to record the information includes
a. A debit to bargain purchase gain on acquisition of P2,000,000
b. A debit to liabilities of P2,000,000
c. A debit to goodwill of P2,000,000
d. A debit to retained earnings of P2,000,000
Use the following information for 43 and 44:
Ping Company acquires all of Sun Corp. in an asset acquisition. Ping paid P1,000,000 more
than Sun’s book value, and this excess was attributed entirely to goodwill, as all of Sun’s
asset and liabilities were carried at amounts equivalent to fair value. At the time of the
combination, a lawsuit was pending against Sun, which Sun had no recorded on its books. It
was felt at the time that Sun would win the lawsuit, so no provision for it was made when
Ping recorded the asset acquisition.
43. Six months after the acquisition, new information reveals that the expected value of
lawsuit at the date of acquisition was P400,000. The appropriate entry on Ping’s
books record this new information.
a. Retained earnings 400,000
Estimated lawsuit liability 400,000
b. Loss on lawsuit 400,000
Estimated lawsuit liability 400,000
c. Goodwill 400,000
Estimated lawsuit liability 400,000
d. No entry required.

44. Assume the same information as above, except that the value change is a result of
events occurring subsequent to acquisition. The appropriate entry on Ping’s books to
record the new information.
a. Retained earnings 400,000
Estimated lawsuit liability 400,000
b. Loss on lawsuit 400,000
Estimated lawsuit liability 400,000
c. Goodwill 400,000
Estimated lawsuit liability 400,000
d. No entry required

Use the following information for 45 to 47:


Netcom acquires all the assets of P570,000,000 and liabilities amounting to P100,000,000 of
Unicom by issuing 25,000,000 shares of no-par common stock valued at P400,000,000 plus
cash of P50,000,000 and records the acquisition as a statutory merger acquisition. Included
in the agreement is a contingency guaranteeing the former shareholders of Unicom that
Netcom’s shares will be worth at least P350,000,000 after one year. If not, Unicom will issue
additional shares to bring the total value of shares issued to P350,000,000. This contingency
is valued at P20,000,000 at the date of acquisition. At the end of the first year following the
acquisition, the 25,000,000 shares of Netcom’s stock held by the former shareholders of
Unicom are worth P12/share.
45. The Netcom’s journal entry to initially record the acquisition.
a. Investment in S 470,000,000
Common stock 400,000,000
Cash 50,000,000
PIC – stock contingency 20,000,000
b. Assets 570,000,000
Liabilities 100,000,000
Common stock 400,000,000
Cash 50,000,000
PIC – stock contingency 20,000,000
c. Loss on stock contingency 470,000,000
Common stock 400,000,000
Cash 50,000,000
PIC – stock contingency 20,000,000
d. No entry required.

46. How many additional shares must Netcom subsequently issue to the former
shareholders of Unicom?
a. 25,000,000 c. 2,083,333
b. 4,166,667 d. No additional shares

47. The Netcom’s journal entry to record the issuance of the additional shares in the
previous number should be:
a. Loss on stock contingency 50,000,000
Common stock 50,000,000
b. PIC – stock contingency 20,000,000
Loss on stock contingency 30,000,000
Common stock 50,000,000
c. PIC – stock contingency 20,000,000
PIC – others 30,000,000
Common stock 50,000,000
d. No entry required.

48. Polk issues common stock to acquire all the assets of the Sam Company on January 1,
20x5. There is a contingent share agreement, which states that if the income of the
Sam Division exceeds a certain level during 20x5 and 20x6, additional shares will be
issued on January 1, 20x7. The impact of issuing the additional shares is to
a. increase the price assigned to fixed assets
b. have no effect on asset values, but no reassign the amounts assigned to equity
accounts
c. reduce retained earnings
d. record additional goodwill

49. P Corporation issued 10,000 shares of common stock with a fair value of P25 per
share for all the outstanding common stock of S Company in a business combination
properly accounted for as an acquisition. The fair value of S Company’s net assets on
that date was P220,000. P Company also agreed to issue an additional 2,000 shares
of common stock with a fair value of P50,000 to the former stockholders of S
Company as an earnings contingency. Assuming that the contingency is expected to
be met, the P50,000 fair value of the additional shares to be issued should be treated
as a(n)
a. decrease in noncurrent liabilities of S Company that were assumed by P Company
b. decrease in consolidated retained earnings
c. increase in consolidated goodwill
d. decrease in consolidated other contributed capital

50. P Co. issued 5,000 shares of its common stock, valued at P200,000, to the former
shareholders of S Company two years after S Company was acquired in an all-stock
transaction. The additional shares were issued because P Company agreed to issue
additional shares of common stock if the average past combination earnings over the
next two years exceeded P500,000. P Company will treat the issuance of the
additional shares as a (decrease in)
a. retained earnings
b. goodwill
c. paid-in capital
d. non-current liabilities of S Company assumed by P Company
Use the following information for 51 to 54:
Bullen Inc. acquired assets and liabilities of Vicker Inc on January 1, 20x4. The book value
and fair value of Vicker’s accounts on that date (prior to creating the combination) follow,
along with the book value of Bullen’s accounts:
Bullen Vicker Vicker
Item Book Value Book Valuw Fair Value
Retained Earnings 1/1/x4 …………. P160,000 P240,000
Cash and Receivables ……………….. 170,000 70,000 70,000
Inventory ………………………………….. 230,000 170,000 210,000
Land ………………………………………….. 280,000 220,000 240,000
Buildings (net) …………………………. 480,000 240,000 270,000
Equipment (net) ……………………… 120,000 90,000 90,000
Liabilities …………………………………. 650,000 430,000 420,000
Common Stock ………………………… 360,000 80,000
Additional paid-in capital ……….. 20,000 40,000

51. Assume that Bullen issued 12,000 shares of common stock with a P5 par value and a
P47 fair value to obtain all of Vicker’s outstanding stock. In this transaction how
much goodwill should be recognized?
a. P144,000 d. P60,000
b. P104,000 e. P 0
c. P 64,000

52. Assume that Bullen issued 12,000 shares of common stock with a P5 par value and a
P42 fair value for all the outstanding shares of Vicker. What will be the Additional
Paid-In Capital and Retained Earnings after the combination?
a. P20,000 and P160,000 d. P464,000 and P160,000
b. P20,000 and P260,000 e. P380,000 and P260,000
c. P380,000 and P160,000

53. Assume that Bullen issued preferred stock with a par value of P240,000 and a fair
value of P500,000 for all of the net assets of Vicker in a business combination. What
will be the balance in the Inventory and Land accounts after the business
combination?
a. P440,000, P496,000 d. P402,000, P520,000
b. P440,000, P520,000 e. P427,000, P510,000
c. P425,000, P505,000

54. Assume that Bullen paid a total of P480,000 in cash for all the shares of Vicker. In
addition, Bullen paid P35,000 to a group of attorneys for their work in arranging the
combination to be accounted for as an acquisition. What will be the balance in
goodwill?
a. P 0 c. P35,000
b. P20,000 d. P55,000

55. Prior to being united in a business combination, AA, Inc., and WS Corporation had he
following stockholders’ equity figures:
AA WS
Common Stock (P1 par value) P180,000 P45,000
Additional paid-in capital 90,000 20,000
Retained earnings 300,000 110,000

AA issues 51,000 new shares of its common stock valued at P3 per share for all the
outstanding stock of WS. Assume that AA acquires WS. Immediately afterward,

What are the Additional Paid-In Capital and Retained Earnings, respectively?
a. P104,000 and P300,000 c. P192,000 and P300,000
b. P110,000 and P410,000 d. P212,000 and P410,000

56. Pat Corporation paid P100,000 cash for the net assets of Sag Company, which
consisted of the following:
Book Value Fair Value
Current Assets P40,000 P56,000
Plant and equipment (net) 160,000 220,000
Liabilities assumed (40,000) (36,000)

Assume Sag Company is dissolved. The plant and equipment acquired in this
business combination should be recorded at:
a. P220,000 c. P183,332
b. P200,000 d. P180,000

57. Balter Inc. acquired Jersey Company on January 1, 20x4. When the purchase
occurred Jersey Company had the following information related to fixed assets:

Land P80,000
Building 200,000
Accumulated Depreciation (100,000)
Equipment 100,000
Accumulated Depreciation (50,000)

The building has a 10-year remaining useful life and the equipment has a 5-year
remaining useful life. The fair values of the assets on that date were:
Land P100,000
Building 130,000
Equipment 75,000
What is the 20x4 depreciation expense Balter will record related to purchasing Jersey
Company?
a. P 8,000 c. P28,000
b. P15,000 d. P30,000
Use the following information to questions 58 and 59:
North Company issued 24,000 shares of its P20 par value common stock for the net assets of
Prairie Company in business combination under which Prairie Company will be merged into
North Company. On the date of the combination, North Company common stock had a fair
value of P30 per share. Balance sheets for North Company and Prairie Company
immediately prior to the combination were as follows:

North Praine
Current Assets P1,314,000 P192,000
Plant and Equipment (net) 1,725,000 408,000
Total P3,039,000 P600,000

Liabilities P900,000 P150,000


Common Stock, P20 par value 1,650,000 240,000
Other Contributed Capital 218,000 60,000
Retained Earnings 271,000 150,000
Total P3,039,000 P600,000

58. If the business combination is treated as an acquisition and Prairie Company’s net
assets have a fair value of P686,400, North Company’s balance sheet immediately
after the combination will include goodwill of
a. P30,600. c. P33,600.
b. P38,400. d. P56,000.

59. If the business combination is treated as an acquisition and the fair value of Prairie
Company’s current assets is P270,000, its plant and equipment is P726,000, and its
liabilities are P168,000, North Company’s financial statements immediately after the
combination will include:
a. Negative goodwill of P108,000.
b. Plant and equipment of P2,133,000.
c. Plant and equipment of P2,343,000.
d. An ordinary gain of P108,000.

60. Publics Company acquired the net assets of Citizen Company during 20x4. The
purchase price was P800,000. On the date of the transaction, Citizen had no long-
term investments in marketable equity securities and P400,000 in liabilities. The fair
value of Citizen assets on the acquisition date was as follows:

Current assets…………………………………………………. P800,000


Noncurrent assets…………………………………………… 600,000

How should Publics account for the P200,000 difference between the fair value of
the net assets acquired, P1,000,000, and the cost, P800,000?
a. Retained earnings should be reduced by P200,000.
b. Current assets should be recorded at P685,000 and noncurrent assets recorded at
P515,000.
c. A P200,000 gain on acquisition of business should be recognized.
d. A deferred credit of P200,000 should be set up and subsequently amortized to
future net income over a period not to exceed 40 years.
Use the following information to 61 to 62:
During its inception, Devon Company purchased land for P100,000 and a building for
P180,000. After exactly 3 years, it transferred these assets and cash of P50,000 to a newly
created subsidiary. Regan Company, in exchange for 15,000 shares of Regan’s P10 par value
stock. Devon uses straight-lie depreciation. Useful life for the building is 30 years, with zero
residual value.
61. At the time of the transfer, Regan Company should record:
a. Building at P180,000 and no accumulated depreciation
b. Building at P162,000 and no accumulated depreciation.
c. Building at P200,000 and accumulated depreciation of P24,000
d. Building at P180,000 and accumulated depreciation of P18,000

62. Regan Company will report:


a. Additional paid-in capital of P0
b. Additional paid-in capital of P150,000
c. Additional paid-in capital of P162,000
d. Additional paid-in capital of P180,000

63. The Geek Company acquired net assets of the Okay Company for a consideration
transferred of P112 million. Ta the acquisition date, the carrying amount of Okay’s
net assets was P100 million and their fair value was P120 million.

How should the difference between the consideration transferred and the net assets
acquired be presented in Geek’s financial statement, according to PFRS 3 Business
combination?
a. Gain on bargain purchase of P8 million recognized in other comprehensive
income.
b. Gain on bargain purchase of P8 million deducted from other intangible assets.
c. Gain on bargain purchase of P8 million recognized in profit or loss.
d. Goodwill of P12 million as an intangible assets.

64. Homer Ltd. Is seeking to expand its share of the Widgets market and has a negotiable
to take over the operation of Tan Ltd. on January 1, 20x4. The balance sheets of the
two companies as of December 31, 20x4 were as follows:

Homer Tan
Cash P23,000 P12,000
Receivables 25,000 34,700
Inventory 35,000 27,600
Freehold Land 150,000 100,000
Building (net) 60,000 30,000
65,000 46,000
Goodwill 25,000 2,000
P383,500 P252,300

Accounts payable P56,000 P43,500


Mortgage loan 50,000 40,000
Debentures 100,000 50,000
Common stock, 100,000 shares 100,000
Common stock, 60,000 shares 60,000
Additional paid-in capital 28,500 26,800
Retained earnings 49,000 32,000
P383,500 P252,300

Homer Ltd. Is to acquire all the assets, except cash of Tan Ltd. The assets of Tan are
all recorded at fair value except:

Fair Value
Inventory………………………………………………. P39,000
Freehold Land……………………………………….. 130,000
Buildings………………………………………………. 40,000

In exchange, Homer Ltd. Is to provide sufficient extra cash to allow Tan Ltd. to repay
all of its outstanding debts and its liquidation costs of P2,400, plus two fully paid
shares in Homer Ltd. for every three shares held in Tan Ltd. The fair value of a share
in Hastings Ltd. is P3.20. An investigation by the liquidator of Tan Ltd. reveals that on
December 31, 20x3, the outstanding debts were outstanding but nit have been
recorded:

Accounts payable……………………………………………. P1,600


Mortgage interest…………………………………………… 4,000

The debentures issued by Tan Ltd. are to be redeemed at 5% premium. Cost of


issuing the shares are P1,200.
The excess of fair value of net assets over cost or gain on acquisition that will be
recognized immediately in the income statement is:
a. Nil or Zero c. P29,700
b. P17,700 d. P34,300
Use the following information for questions 65 and 66:
ACME Co. paid P110,000 for the net assets of Comb Corp. At the time of the acquisition the
following information was available related to Comb’s balance sheet:

Book Value Fair Value


Current Assets P50,000 P50,000
Building 80,000 100,000
Equipment 40,000 30,000
Liabilities 30,000 30,000
65. What is the amount recorded by ACME for the Building?
a. P110,000 c. P80,000
b. P 20,000 d. P100,000

66. What amount of gain (loss) on disposal of a business should Comb Corp. recognize?
a. Gain of P30,000 c. Loss of P30,000
b. Gain of P60,000 d. Loss of P60,000
Use the following information for 67 to 72:
TT Corporation acquired assets and assumed liabilities of SS Corporation’s on December 31,
20x4. Balance sheet data for the two companies immediately following the acquisition
follow:

TT SS
Items Corporation Corporation
Cash P49,000 P30,000
Accounts Receivable 110,000 45,000
Inventory 130,000 70,000
Land 80,000 25,000
Building and Equipment 500,000 400,000
(223, (165,
Less: Accumulated Depreciation 000) 000)
________
Investment in SS Corporation Stock 198,000 ___
Total Assets P844,000 P405,000
Accounts Payable 61,500 28,000
Taxes Payable 95,000 37,000
Bonds Payable 280,000 200,000
Common Stock 150,000 50,000
Retained Earnings 257,500 90,000
Total Liabilities and Stockholders'
Equity P844,000 P405,000

At the date of the business combination, the book values of SS’s net assets and liabilities
approximated fair value except for inventory, which had a fair value of P85,000, and land,
which had a fair value of P45,000. Indicate the appropriate total that should appear in the
balance sheet prepared immediately after the business combination.
67. What amount of inventory will e reported?
a. P70,000 c. P200,000
b. P130,000 d. P215,000

68. What amount of goodwill will be reported?


a. P-0- c. P43,000
b. P23,000 d. P58,000

69. What amount of total assets will be reported?


a. P84,400 c. P1,109,000
b. P1,051,000 d. P1,249,000

70. What amount of total liabilities will be recorded?


a. P265,000 c. P701,500
b. P436,500 d. P1,249,000

71. What amount of retained earnings will be reported?


a. P547,500 c. P347,500
b. P397,500 d. P257,500

72. What amount of total stockholders’ equity will be reported?


a. P407,500 c. P844,000
b. P547,500 d. P1,249,000
Use the following information for 73 and 74:
AA Company acquired all of the BB Corporation’s assets and liabilities on October 2, 20x5, in
a business combination at that date. BB reported assets with a book value of P1,198,080 and
liabilities of P683,520. AA noted that BB included the amount of P76,800 obsolete
merchandise at the acquisition date that did not appear of any value. AA also determined
that an old delivery van previously used by BB had a fair value of P230,400, but had not
been recorded by BB. Except for machinery and equipment, AA determined the fair value if
all other assets and liabilities reported by BB approximated the recorded amounts. In
recording the transfer of assets and liabilities in its books. AA recorded a gain on acquisition
of P178,560. AA paid P392,640 to acquire BB’s assets and liabilities.
73. If the books value of BB’s machinery and equipment was P414,720, what was their
fair values?
a. Nil c. P394,560
b. P322,080 d. None of the above

74. Assuming that BB recorded goodwill of P482,400. AA paid P1,244,400 to acquire BB’s
assets and liabilities. If the book value of the machinery and equipment was
P619,800, what was their FV?
a. Nil c. P790,320
b. P713,640 d. None of the above
Use the following information to 75 to 77:
On September 18, 20x5, XX Co. acquired all the YY Inc.’s P2,580,000 identifiable assets and
P636,000 liabilities. Carrying amounts of the YY”s assets and liabilities equal their fair value
except for the overvalued furniture and fixtures.
 As a consideration, XX issued its own shares with a market value of P2,058,000
and cash amounting to P450,000.
 Contingent consideration that was probable and reasonably estimated on the date of
acquisition amount to P177,600.
 The merger resulted into P776,400 goodwill.
 Assuming XX had P5,868,000 total assets and P3,277,200total liabilities prior to the
combination and no additional cash payments were made, but expenses were
incurred for related cost amounting to P33,600.

75. Determine the amount of overvaluation of the furniture and fixtures.


a. Nil c. P34,800
b. P33,600 d. None of the above

76. After the merger, how much is the combined total identifiable assets in the books of
the acquirer?
a. Nil c. P7,963,200
b. P6,644,400 d. None of the above

77. After the merger, how much is increase in liabilities in the books of the acquirer?
a. Nil c. P880,800
b. P847,200 d. None of the above

78. .On August 1,20x7, The ReSA Co. acquired the net assets of Lv Co. for a consideration
of P20,940,000 cash. On acquisition date, the carrying amount of LV’s net assets was
P14,320,000 and temporary appraisal of P14,862,000 was attributed to the net
assets. In addition to the consideration transferred above is another P1,218,000 cash
to be transferred nine months after the acquisition date is specified profit target was
met by the acquirer.

On the acquisition date, there was only a low probability of the profit target met so
their value of the additional consideration liability was determined to be P561,600.
On December 31, 20xx7, an update of the provisional fair value of P20,178,000 was
attributed to the net assets. Also, at year end the estimated amount of the
consideration liability is determined to decrease by P86,400 from the last date of the
change in estimate.
On March 31, 20x8, the estimated amount of the consideration liability is determined
to be probable at P340,800. On July 1, 20x8 the temporary appraisal decrease by
P1,128,000 from the last additional valuation date.

The provisional value was finalized on August 31, 20x8 with an amount that us higher
by P1,284,000 from the temporary appraisal as of July1, 20x8i.

As a subsequent event, the profit target was met and the P1,218,000 was
transferred. What amount of goodwill (gain) is presented in the separate statement
of financial position of the acquirer as of December 31, 20x8?

a. P39,600 c. P2,451,600
b. P1,323,600 d. P7,296,000
Use the following information for 78 and 79:
Purpoise Corporation acquired Sims Company through an exchange of common shares. All of
Sims assets and liabilities were immediately transfers to Purpoise. Purpoise Company’s
common stock was trading at P20 per share at the time of exchange. The following selected
information for Purpoise Corporation is also available:
Before Acquisition After Acquisition
Par value of share outstanding ……… P200,000 P250,000
Additional Paid in capital ………………. 350,000 550,000
79. What is the fair value of Porpoise’s common stocks?
a. P10 c. P4
b. P5 d. P1

80. What is the fair value of Sim’s net assets, if goodwill of P56,000 is recorded?
a. P194,000 c. P300,000
b. P244,000 d. P306,000

81. Chapel Hill Company had common stock of P350,000 and retained earnings of
P490,000. Blue Town Inc. had common stock of P700,000 and retained earnings of
P980,000. On January 1,20x4, Blue Town issued 34,000 shares of common stock with
a P12 par value and a P35 fair value of all of chapel Hill Company’s net assets. This
combination was accounted for as an acquisition. Immediately after the combination,
what were the consolidated net assets?
a. P2,870,000 d. P1,680,000
b. P2,520,000 e. P1,190,000
c. P2,030,000

82. Vibe Company purchased the net assets of Atlantic Company in a business
combination accounted for as a purchase. As a result, goodwill was recorded. For tax
purposes, this combination was considered to be a tax-free merger. Included in the
assets is a building with an appraised value of P210,000 on the date of the business
combination. This asset had a net book value of P70,000, based on the use of
accelerated depreciation for accounting purposes. The building had an adjusted tax
basis to Atlantic (and to Vibe as a result of the merger) of P120,000. Assuming a 36%
income tax rate, at what amount should Vibe record this building on its books after
the purchase?
a. P120,000 c. P140,000
b. P134,400 d. P210,000
Use the following information for 82 to 86:
Zyxel Corporation acquired all the assets and liabilities of Globe Tattoo Corporation by
issuing shares of its common stock On January 1, 2011. Partial balance sheet data for the
companies prior to the business combination and immediately following the combination is
provided:

Zyxel Globe Tattoo


Book Value Book Value Combination
Cash P65,000 P25,000 P90,000
Accounts Receivable 72,000 20,000 94,000
Inventory 33,000 45,000 88,000
Building and equipment (net) 400,000 150,000 650,000
__________ __________ __________
Goodwill _ _ ?
Total Assets P570,000 P240,000 P?

Accounts Payable P50,000 P25,000 P75,000


Bonds Payable 250,000 100,000 350,000
Common stock, P2 par 100,000 25,000 160,000
Additional paid-in capital 65,000 20,000 245,000
Retained earnings 105,000 70,000 ?
Total Liabilities and
Equities P570,000 P240,000 P?

83. What number of shares did Zyxel issue for this acquisition
a. P80,000 c. P30,000
b. P50,000 d. P17,500

84. At what price was Zyxel stock was issued for this acquisition?
a. P2.00 c. P6.00
b. P50.00 d. P8,00

85. What was the fair value of the net assets held by Globe Tattoo at the date of
combinations?
a. P115,000 c. P270,000
b. P227,000 d. P497,000

86. What amount of goodwill will be reported by the combined entity immediately
following combination?
a. P13,000 c. P173,000
b. P125,000 d. P413,000

87. What balance in retained earnings will the combined entity report immediately
following the combinations?
a. P35,000 c. P105,000
b. P70,000 d. P175,000

88. Bats Inc., a new corporation formed and organized because of the recent
consolidation of II Inc. and JJ. Inc., shall issue 10% participating preferred stocks with
a par value of P100 for II and JJ net assets contributions, and common shares with a
par value of P50 for the difference between t he total shares to be issued and the
preferred shared to be issued. The total shares to be issued by Bats shall be
equivalent to average annual earnings capitalized at 10%. Relevant data on II and JJ
follows:

II JJ
Total Assets …………………………………………. P720,000 P921,600
Total Liabilities …………………………………….. 432,000 345,600
Annual Earnings (average) …………………… 46,000 69,000

The total preferred shares to be issued and the amount of goodwill to be recognized by Bats
are:
a. Preferred shares: 8,640 Goodwill: P288,000
b. Preferred shares: 5,760 Goodwill: P288,000
c. Preferred shares: 2,880 Goodwill: P864,000
d. Preferred shares: 7,280 Goodwill: P864,000

THEORIES
TRUE OR FALSE
1. When two entities competing in the same industry combine. It is called a horizontal
business combination
2. Horizontal business combinations are likely to occur when management is attempting to
dominate a geographic segment of the market
3. One way that a horizontal business combination can increase sales for an entity is to
expand into new product markets
4. A vertical business combination generally involves companies attempting to improve the
efficiency of operations by purchasing suppliers inputs or purchasers of outputs
5. When a retail clothing store purchases a competitor in another city, a vertical combination
has occurred
6. Vertical combination is one where the entities have a potential buyer seller relationship
7. A business combination in which a supplier of raw materials is acquired is a conglomerate
combination
8. A conglomerate combination is often undertaken to help increase income stability due to
diversifying the asset base of an entity
9. Conglomerate combination are easy for the government to challenge in court
10. If negotiation between management groups leads to a mutually agreeable business
combination, the process is called a friendly takeover
11. An offer by an acquirer to buy the stock of another company is commonly called tender
offer
12. A tender offer that is opposed by the acquire management is called a hostile bid
13. Greenmail exist when a company is encourage to buy a potential acquire
14. A poison pill is the term used to describe the issuance of a special kind of convertible
preferred stock to deter the acquisition of the company
15. The sale of the crown jewel defensive maneuver involves the sale of more assets than
does the scorched earth defense
16. The fatman defensive maneuver involved the acquisition of assets by the potential
acquire
17. Golden parachutes give a bonus to all employees if the company is acquired
18. The packman defensive maneuver is where a potential acquire attempts to purchase the
acquirer
19. A business combination occurs when one entity gains control over the net assets of
another entity
20. The only way to attain control over the net assets of another entity is to purchase the net
assets
21. In an acquisition where the acquirer pays cash for the acquire assets, the book value of
the acquirer increases
22. In an acquisition of assets for assets, the ownership structure of the acquiree does not
change
23. In an acquisition of assets for assets, the ownership structure of the acquirer change
24. There is an increase in the total capitalization of an acquirer when the acquirer issues
stock for acquire assets
25. In an exchange of stock (acquirer) for assets( acquire), the ownership structure of the
acquire does not change
26. In an exchange of stock (acquirer) for assets( acquire), the acquire stockholders become
acquirer stockholders
27. Control over the acquiree assets is directly achieved in an asset for asset exchange but
indirectly achieved in an asset (acquirer) for stock (acquire) exchange
28. A business combination that occurs where only one of the original entities in existence
after the combination is called a statutory consolidation
29. The acquire entity is liquidated in a statutory merger
30. For a business combination to qualify as a statutory consolidation, a new corporation
must be formed
31. In a statutory consolidation form of business combination, the Retained Earnings
account of the newly formed corporation has a balance of zero immediately after the
combination
32. After completing a business combination in the form of a statutory merger or statutory
consolidation, there is only one legal entity in existence
33. In a business combination accomplished as a stock acquisition normally two companies
exist after the combination
34. In a business combination accomplished as a stock acquisition must be accomplished
with a stock for stock exchange
35. A stock acquisition is the only form of business combination that might requires the
preparation of consolidated financial statements.
36. The substance of statutory mergers, statutory consolidations, and stock acquisitions is
the same if income tax considerations are ignored
37. There are no uncertainties when two companies agree on a business combination
38. When the acquisition price of an acquire is contingent on acquire future earnings, the
acquisition price may be change
39. When the acquisition price of an acquire is contingent on the market value of the
acquirer stock, the acquisition price may be change
40. For business combinations to qualify as reorganizations ( for tax purpose), the acquire
stockholders must receive voting common stock of the acquirer
41. There are different required levels of stock ownership in the acquire for the three
different types of reorganizations for tax purposes
42. One important benefit in a business combination is any net operating loss carry forward
that might exist and be available to the acquirer.
43. Which of the following types of business combinations typically occurs when
management is attempting to monopolize a particular industry?
a. Horizontal combination
b. Vertical combination
c. Conglomerate combination
d. Market domination can be the goal of any type of combination
44. Horizontal business combinations occur when one entity purchases which of the
following?
a. A supplier
b. A customer
c. A competitor
d. None of the above
45. Horizontal business combinations help sales increase by all but which of the following?
a. Entering new product markets
b. Taking control of a distribution system
c. Increasing production capacity
d. Expanding into new geographic regions
46. Which of the following types of business combinations typically occurs when
management is attempting to improve the efficiency of operations?
a. Horizontal combination
b. Vertical combination
c. Conglomerate combination
d. Improve efficiency can be the goal of any type of combination
47. A vertical combination occur when one entity acquires another which has the following
characteristic(s)?
a. The acquiree purchases the acquirer’s outputs
b. The acquire is a competitor of the acquirer
c. The acquire supplies raw materials to the acquirer
d. Either a or c
48. Which of the following is a vertical combinations?
a. A combination where the two entities unrelated
b. A combination where the two entities are competitors in the same industry
c. A combination where the two entities have a potential buyer/seller relationship
d. None of the above describes a vertical combination
49. Which of the following types of business combinations typically occurs when
management is attempting to diversify its investment?
a. Horizontal combination
b. Vertical combination
c. Conglomerate combination
d. Diversification can be the goal of any type of combination
50. Management acquires a business in a tangentially related industry to the current
business. What form of business combination is accomplish?
a. Vertical combination
b. Conglomerate combination
c. Mega combination
d. Horizontal combination
51. One reason for conglomerate combinations is that management has become more
aware that it helps accomplish which of the following?
a. It helps increase income stability provided by diversifying the asset base of an entity
b. It helps to increase market share in the industry
c. It helps assure a constant supply of raw materials
d. A conglomerate combination helps accomplish all three
52. Business combinations that result in one dominant company in an industry are said to
have formed which of the following?
a. Pure competition
b. Monopoly
c. Oligopoly
d. Free market
53. The business enterprises that inter into a business combinations are termed the:
a. Merging companies
b. Joining companies
c. Constituent companies
d. Combiner companies
54. When an offer is made to acquire a company and the acquire management supports the
offer is called which of the following?
a. Friendly takeover
b. Tender offer
c. Hostile takeover
d. Defensive measure
55. The defensive maneuver where a company buys stock from a potential acquirer of a
premium over the market price is called which of the following?
a. White knight
b. Shark repellent
c. Greenmail
d. Sale of the crown jewels
56. The defensive maneuver where a company seeks to be acquired by a company perceived
to be a better match than the company making an offer to buy the potential acquire is
called which of the following?
a. Poison pill
b. White knight
c. Golden Parachutes
d. Pac-man defense
57. Company A makes a hostile take-over bid for control of Company B. In response,
Company B makes a counter-offer to purchase shares from Company A’s shareholders.
Which of the following best describes Company B’s response?
a. Pac-man defense
b. Selling the crown jewels
c. Poison pill
d. A Hostile defense
58. Company A has made an offer to purchase all of the outstanding shares of Company B
for P10 per share (the current market value of the share). In response to Company A’s offer,
the shareholders of Company B were given rights to purchase additional shares at P8 per
share. Which of the following tactics was employed by Company B to prevent Company A
from acquiring control of Company B?
a. Pac-man defense
b. Selling the crown jewels
c. Poison pill
d. A Reverse take-over
59. What is the term used for the defensive maneuver where management of a potential
acquiree sells desirable assets to reduce the company’s value?
a. Sale of the crown jewels
b. Scorched earth defense
c. Pac-man defense
d. Greenmail
60. Shark repellent is a term for administration measures that may make a hostile take-over
more difficult. Which of the following is not a form of shart repellent?
a. Staggering board of director terms
b. Residency requirement for board members
c. Issuance a convertible preferred stock that converts into common stock of the acquirer if a
takeover is accomplished
d. A supermajority vote is required to approve an acquisition
61. Defensive maneuver can be internal to the potential acquiree (management or
stockholders) or may involve activities external to the acquiree. Which of the following is not
an internal defensive maneuver?
a. Residency requirement to board members
b. Golden Parachutes
c. Pac-man defense
d. A supermajority vote is required to approve an acquisition
62. Able ltd. offers to buy shares from the existing shareholders of Wei Co. at a premium
price. The current management and board of directors of Wei have let the Wei shareholders
know that they do not approve of this. This is an example of a(n):
a. open market purchase
b. hostile takeover
c. poison pill strategy
d. reverse takeover
63. Control over an acquiree can be attained through which of the following?
a. Acquisition of the acquiree assets
b. Acquisition of the acquire stock
c. Either acquisition of the acquiree assets or stock
d. Neither acquisition of the acquiree assets or stock
64. In an acquisition of assets, the acquirer must give up which of the following?
a. Cash
b. Other assets
c. Liabilities
d. Any of the above can be given
65. In an acquisition where there is an exchange of assets for assets, how does the value of
the acquire net assets change?
a. The net assets increase
b. The net assets decrease
c. There is no change in net assets
d. The net assets may increase, decrease or remain the same
66. In an acquisition where there is an exchange of assets for assets, how does the
ownership structure of acquiree change?
a. There is no change on the acquiree ownership structure
b. The acquirer stockholders become the acquiree stockholders
c. The acquirer and acquiree stockholders share ownerdhip of the acquiree
d. It is not possible to determine if there is a change in the acquiree ownership structure
67. In an acquisition where there is an exchange of assets for assets, how does the
ownership structure of acquirer change?
a. There is no change on the acquirer ownership structure
b. The acquirer stockholders become the acquiree stockholders
c. The acquirer and acquiree stockholders share ownerdhip of the acquiree
d. It is not possible to determine if there is a change in the acquiree ownership structure
68. In an acquisition where there is an exchange of stock (acquirer) for assets (acquiree),
how does the value of the acquiree net assets change?
a. The net assets increase
b. The net assets decrease
c. There is no change in net assets
d. The net assets may increase, decrease or remain the same
69. In an acquisition where there in an exchange of stock (acquirer) for assets (acquiree).
How does the ownership structure of the acquiree change?
a. There is no change on the acquirer ownership structure
b. The acquirer stockholders become the acquiree stockholders
c. The acquirer and acquiree stockholders share ownerdhip of the acquiree
d. It is not possible to determine if there is a change in the acquiree ownership structure
70. In an acquisition where there in an exchange of stock (acquirer) for assets (acquiree).
How does the ownership structure of the acquirer change?
a. There is no change on the acquirer ownership structure
b. The acquiree (company) becomes stockholders of the acquirer
c. The acquireee stockholders as individuals become owners of the acquirer
d. It is not possible to determine if there is a change in the acquirer ownership structure
71. Control over acquiree assets is attained in a business combination, Indirect control is
attained in which type of exchange?
a. Assets for assets
b. Stock (acquirer) for assets (acquiree)
c. Stock for stock
d. Either b or c
72. Which of the following forms of business combination is not subject to laws specific to
business combinations?
a. Asset for asset acquisition
b. Statutory merger
c. Statutory consolidation
d. All three are subject to laws
73. Which of the following is not true statement with regard to a statutory merger?
a. One entity continues to exist
b. One entity ceases to exist
c. The name of the new entity is not the same as either of the entities
d. All of the above are true statements with regard to a statutory merger
74. Which of the following is not true statement with regard to a statutory consolidation
form of business combination?
a. A new corporation must be formed
b. Control of the net assets of the combining entities must be acquired by the new entity
c. The net assets of the combining entities must be acquired with assets of the new
corporation
d. The combining entities both cease to exist after the combination
75. Following the completion of a business combination in the form of a statutory
consolidation, what is the balance in the new corporation’s Retained Earnings account?
a. The acquirer Retained Earnings account balance
b. The acquiree Retained Earnings account balance
c. Zero
d. The sum of the acquirer and acquiree Retained Earnings account balance
76. Which if the following is not true with regard to business combination accomplished in
the form of a stock acquisition?
a. Two companies remain in existence after the combination
b. A parent- subsidiary relationship is said to exist
c. Consolidated financial statements are normally required
d. All of the above statements are true
77. Which of the following contingencies may change the cost of an acquisition?
a. Future acquiree earnings
b. Future value of acquiree stock
c. Future value of acquirer stock
d. Future value of acquirer debt
78. To qualify as a reorganization (for tax purpose), a business combination must meet which
of the following criteria?
a. Acquiree stockholders continue an indirect ownership interest in the acquiree
b. The acquirer must continue the acquiree business or employ a significant portion of the
acquiree net assets in an ongoing business
c. The combination must be for a valid business purpose
d. All of the above criteria are required for a combination to qualify as a reorganization
79. Which of the following is not a business combination?
a. Statutory amalgamation
b. Joint venture
c. A company’s purchase of 100% of another company’s net assets
d. A company’s purchase of 80% of another company’s voting shares
80. Under PFRS 3, Business Combinations, which method must be used to account for
balances combinations?
a. Purchase method
b. Pooling-of-interest method
c. Acquisition method
d. New entity method
81. After an exchange of shares In a business combination, each group of shareholders held
50% of the voting rights. Which of the following factors should be considered in determining
the acquirer?
a. Head office location
b. Composition of the board of directors
c. If there are material transaction between the combining companies
d. Which company initiated the combination
82. Perez Co. plans acquire Roo Co. has substantial depreciable assets that have fair values in
excess of their book values. Considering only the income tax impact, which of the following
statement is true?
a. Perez would prefer to purchase Roo’s assets and Roo would prefer to sell its shares to
Perez
b. Perez would prefer to purchase Roo’s assets and Roo would prefer to sell its assets to
Perez
c. Both Perez and Roo would prefer Perez to purchase Roo’s shares
d. Both Perez and Roo would prefer Perez to purchase Roo’s assets
83. Perez Co, acquired Roo Co. in a business combination, Roo issued new shares to Perez
shareholders in exchange for their outstanding shares. What type of share exchange is this?
a. Direct exchange
b. Indirect exchange
c. Hostile takeover
d. Reverse takeover
84. Perez Co, acquired Roo Co. in a business combination, Perez issued new shares to Roo’s
shareholders in exchange for their outstanding shares. What type of share exchange is this?
a. Direct exchange
b. Indirect exchange
c. Hostile takeover
d. Reverse takeover
85. Ha Ltd. and Hee Ltd. exchange shares in a business combination, After the share
exchange, each company held the same number of voting shares. Which of the following
statements is true?
a. The company with the highest net assets is consider the acquirer
b. The companies must ask the court to decide which company is the acquirer
c. A number of factors must be considered to determine which company is the acquirer
d. There is no acquirer as this is not a proper business combination
86. How should the transaction costs of issuing shares in an acquisition be recognized?
a. Expensed
b. Capitalized as part of the cost of the shares
c. Deducted in total from shareholder’s equity
d. Deducted from shareholder’s equity, net of related income tax benefits
87. How should the cost of issuing debt in an acquisition be recognized?
a. Expensed
b. Amortized over the term of the debt
c. Deducted the value of the debt
d. Deducted from shareholder’s equity
88. How should accounting fees for an acquisition be treated?
a. Expensed in the period of acquisition
b. Capitalized as part of the acquisition cost
c. Deferred and amortized
d. Deferred until the company is disposed or wound-up
89. Which of the following is not a reason why a private enterprise may be acquired as a
bargain purchase?
a. It is a family business and the next generation does not want to continue the business
b. The owner has health problems and does not have a successor
c. The business only has equity financing and has no debt financing
d. The owner is no longer interested in the business
90. Which of the following statements about a bargain purchase is true?
a. It is reported on the financial statements as an “excess of fair value over cost of assets
acquired”
b. It is reported as a deferred credit on the financial statements called negative goodwill
c. Assets and liabilities of the acquired company are reported at their net book value
d. Assets and liabilities of the acquired company are reported at their fair value
91. What is the most common valuation method used for intangible assets?
a. Market-based
b. Income-based
c. Cost-based
d. Amortized-based
92. How should negative goodwill be shown on the consolidated financial statements of the
acquirer?
a. As a gain on the statement of comprehensive income
b. As a loss on the same of comprehensive income
c. As a liability on the statement of financial position
d. As a separate amount under shareholders’ equity on the statement of financial position
93. Raj. Co. acquired all of Event Ltd.’s common shares. At the date of acquisition, Event had
P80,000 of goodwill resulting from its acquisition of Baker Ltd. a few years ago. At Raj’s date
of acquisition, what is the proper treatment of Event’s P80,000 of goodwill?
a. Event’s goodwill is an identifiable assets and should be included as part of Raj’s purchase
price discrepancy (PPD)
b. Event’s goodwill is an identifiable asset but should not be included as part of Raj’s PPD
c. Event’s goodwill is not an identifiable asset but should be included as part of Raj’s PPD
d. Event’s goodwill is not an identifiable asset and should not be included as part of Raj’s
PPD
94. Which of the following does not constitute a Business Combination under IFRS 3?
a. A Corp purchases the net assets of B Corp
b. A Corp enters into a Joint Venture with b Corp
c. A Corp acquires 51% of B Corp’s voting shares for P1,000,000 in cash
d. A Corp acquires 51% of B Corp’s voting shares for future consideration
95. What is a statutory merger?
a. A merger approved by the Securities and Exchange Commission
b. An acquisition involving the purchase of both stock and assets
c. A takeover completed within one year of the initial tender offer
d. A business combination in which only one company continues to exist as a legal entity
96. A statutory merger is a(n)
a. Business combination in which only one of the two companies continues to exist as a legal
corporation
b. Business combination in which both companies continues to exist
c. Acquisition of a competitor
d. Acquisition of a supplier or a customer
e. Legal proposal to acquire outstanding shares of the target’s stock
97. Liabilities assumed in an acquisition will be valued at the
a. estimated fair value
b. historical book value
c. current replacement cost
d. present value using market interest rates
98. In reference to the IASB disclosure requirements, which of the following is correct?
a. information related to several minor acquisitions may not combined
b. firms are not required to disclose the business purpose for a combination
c. notes to the financial statements of an acquiring corporation must disclose that the
business combination was accounted for by the acquisition method
d. all of the above are correct
99. Goodwill arising from business combination is:
a. charged to Retained Earnings after the acquisition is completed
b. amortized over 40 years or its useful life, whichever is longer
c. amortized over 40 years or its useful life, whichever is shorter
d. never amortized
100. In reference to international accounting to goodwill, which of the following is correct?
a. U.S. companies have complained that past accounting rules foe amortizing goodwill
placed them at a disadvantage in competing against foreign
b. Some foreign countries permitted the immediate write-off of goodwill in stockholders’
equity
c. The IASB and the FASB are working to eliminate differences in accounting for business
combinations
d. All of the above are correct
101. In recording acquisition costs, which of the following procedure is correct?
a. Registration costs are expensed, and not charged against the fair value of the securities
issued
b. Indirect costs are charged against the fair value of the securities issued
c. Consulting fees are expensed
d. None of the above procedure is correct
102. Which of the following statements is incorrect?
a. In an asset acquisition, the books of the acquired company are closed and its assets and
liabilities are transferred to the books of the acquirer
b. In many cases, stock acquisition entail lower total cost than asset acquisition
c. Regulations pertaining to one of the firms do not automatically extend to the entire
merger entity in a stock acquisition
d. A stock acquisition occurs when one corporation pays cash, issues stock, or issues debt for
all or part of the voting stock of another company: and the acquired company dissolves and
ceases to exist as a separate legal entity
103. Which of the following can be used as consideration in a stock acquisition?
a. Cash
b. Debt
c. Stock
d. Any of the above may be used
104. Slocum corporation and Merton Company , both publicly owned companies, are
planning a merger with Slocum being the survivor, which of the following is a requirement
of the merger?
a. The Securities and Exchange Commission must approve the merger
b. The common stockholders of Merton must receive common stock of Slocum
c. The creditors of Merton must approve the merger
d. The board of directors of both Slocum and Merton must receive common stock of Slocum
105. PFRS 3 requires that all the business combinations be accounted for using
a. The pooling of interest method
b. The acquisition method
c. Either the acquisition or the pooling of interests methods
d. Neither the acquisition nor the pooling of interests methods
106. Under the acquisition method if the fair values of identifiable net assets exceed the
value implied by the purchase price of the acquired company, the excess should be
a. accounted for as a goodwill
b. allocated to reduce current and long-lived assets
c. allocated to reduce current assets and classify any remainder as an extraordinary gain
d. allocated to reduce any previously recorded goodwill on the seller’s books and classify any
remainder as an ordinary gain
107. PFRS 3 requires that the acquirer disclose each of the following for each material
business combination except the
a. name and a description of the acquiree acquired
b. percentage of voting equity investment acquired
c. fair value of the consideration transferred
d. each of the above is a required disclosure
108. When the acquisition price of an acquired firm is less than the fair value of the
identifiable net assets, all of the following are recorded at fair value except
a. Assumed liabilities
b. Current Assets
c. Long=lived assets
d. Each of the above is a required disclosure
109. Under PFRS 3
a. both direct and indirect costs are to be capitalized
b. both direct and indirect costs are to be expensed
c. direct costs are capitalized and indirect cost are to be expensed
d. indirect costs are to be capitalized and indirect cost are to be expensed
110. A business combination is accounted for properly as an acquisition, Which of the
following expenses related to effecting the business combination should enter into the
determination of net income of the combined corporation for the period in which the
expense are incurred?
Security issue costs Overhead allocated to the merger
a. Yes Yes
b. Yes No
c. No Yes
d. No No

111. In a business combination, which of the following costs are assigned to the valuation of
the security?
Professional or consulting fees Security issue costs
a. Yes Yes
b. Yes No
c. No Yes
d. No No
112. Parental Company and Sub Company were combined in an acquisition transaction.
Parental was able to acquire Sub at a bargain price. The sum of the fair values of identifiable
assets acquired less the fair value of liabilities assumed exceeded the cost to Parental. After
elimination previously recorded goodwill, there was still “negative goodwill” Proper
accounting treatment by Parental is to report the amount as
a. paid-in capital
b. a deferred credit, which is amortized
c. an ordinary gain
d. an extraordinary gain
113. With an acquisition, direct and indirect expenses are
a. expensed in the period incurred
b. capitalized and amortized over a discretionary period
c. considered a part of the total cost of the acquired company
d. charged to retained earning incurred
114. In a business combination accounted for as an acquisition, how should the excess of fair
value of net assets acquired over the consideration paid be treated?
a. Amortized as a credit to income over a period not to exceed forty years
b. Amortized as a charged to expense over a period not exceed to forty years
c. Amortized directly to retained earnings over a period not to exceed forty years
d. Recorded as an ordinary gain
115. If the value implied by the purchase price of an acquired company exceeds the fair
values of identifiable net assets, the exceeds should be
a. allocated to reduce any previously recorded goodwill and classify any remainder as an
ordinary gain
b. allocated to reduce current and long-lived assets
c. allocated to reduce long-lived assets
d. allocated goodwill
116. P Co. issued 5000 shares of its common stock, valued of P200, 000, to the former
shareholders of S Company two years after S Company acquired in an all stock transaction.
The additional shares were issued because P Company agreed to issue additional shares of
common stock if the average post combination earnings over the next two years exceeded
500,000. P Company will treat the issuance of the additional shares as a (decrease in )?
a. retained earnings
b. goodwill
c. paid-in capital
d. non-current liabilities of S Company assumed by P Company
117. The fair value of assets and liabilities of the acquired entity is to be reflected in the
financial statements of the combined entity, when the acquisition takes place over a period
of time rather than all at once, at what time is tha fair value of the assets and liabilities of
the acquired entity determined?
a. the date the interest in the acquiree was acquired
b. the date the acquirer obtains control of the acquiree
c. the date the acquisition of the largest portion of the interest in the acquiree
d. the date of the financial statement
118. Under PFRS 3, what value of the assets and liabilities is reflected in the financial
statements on the acquisition date of a business combination?
a. Carrying value
b. Fair value
c. Book value
d. Average value
119. What is the appropriate accounting treatment for the value assigned to in-process
research and development acquired in a business combination?
a. Expense upon acquisition
b. Capitalize as an asset
c. Expense if there is no alternative use for the assets used in the research and development
and technological feasibility has yet to be reached
d. Expense until future economic benefits to become certain and then capitalize as an asset
120. An acquired entity has a long-term operating lease for an office building used for
central management. The terms of the lease are very favorable relative to current market
rates. However, the lease prohibits subleasing or any other transfer of rights. In its financial
statements, the acquiring firm should report the value assigned to the lease contract as
a. An intangible asset under the contractual legal criterion
b. A part of goodwill
c. An intangible asset under the separability criterion
d. A building
121. Under PFRS 3 when is a gain recognized in consolidating financial information?
a. When any bargain purchase is created
b. In a combination created in the middle of a fiscal year
c. In an acquisition when the value of all assets and liabilities cannot be determined
d. When the amount of a bargain purchase exceeds the value of the applicable expense
(other than the certain exception) held by the acquired company
122. Company B acquired the net assets of Company S in exchange for cash. The acquisition
price exceeds the fair value of the net assets acquired. How should Company B determine
the amounts to be reported for the plant and equipment, and for long term debt for the
acquired Company S?
Plant and Equipment Long-term Debt
a. Fair value S’s carrying amount
b. Fair value Fair value
c. S’s carrying amount Fair Value
d. S’s carrying amount S’s carrying amount
123. Goodwill represents the excess cost of an acquisition over the
a. sum of the fair values assigned to intangible assets less liabilities assumed
b. sum of the fair values assigned to tangible and identifiable intangible assets acquired less
liabilities assumed
c. sum of the fair values assigned to intangibles acquired less liabilities assumed
d. book value of an acquired company
124. When an acquisition of another company occurs, IASB recommends disclosing all of the
following EXCEPT:
a. goodwill assigned to each reportable segment
b. Information concerning contingent consideration including a description of the
arrangements and the range of outcomes
c. results of operations for the current period if both companies had remained separate
d. a qualitative description of factors that make up the goodwill recognized
125. Separately identified intangible assets are accounted for by amortizing:
a. exclusively by using impairment testing
b. based upon a pattern that reflects the benefits conveyed by the asset
c. over the useful economic life less residual value using the straight-line method
d. over a period not to exceed a maximum of 40 years
126. Acquisition costs such as the fees of accountants and lawyers that were necessary to
negotiate and consummate the purchase are
a. recorded as a deferred asset and amortized over a period not to exceed 15 years
b. expensed if immaterial but capitalized and amortized if over 2% of the acquisition price
c. expensed in the period of the purchase
d. included as part of the price paid for the company purchased
127. Which of the following income factors should not be factored into an estimation of
goodwill?
a. sales for the period
b. income tax expense
c. extraordinary gain
d. cost of goods sold

You might also like